Vous êtes sur la page 1sur 29

No part of this publication may be reproduced or distributed in any form or any means, electronic, mechanical,

photocopying, or otherwise without the prior permission of the author.

GATE SOLVED PAPER


Electrical Engineering
SIGNALS & SYSTEMS
Copyright By NODIA & COMPANY

Information contained in this book has been obtained by authors, from sources believes to be reliable. However,
neither Nodia nor its authors guarantee the accuracy or completeness of any information herein, and Nodia nor its
authors shall be responsible for any error, omissions, or damages arising out of use of this information. This book
is published with the understanding that Nodia and its authors are supplying information but are not attempting
to render engineering or other professional services.

NODIA AND COMPANY

B-8, Dhanshree Tower Ist, Central Spine, Vidyadhar Nagar, Jaipur 302039
Ph : +91 - 141 - 2101150
www.nodia.co.in
email : enquiry@nodia.co.in

GATE SOLVED PAPER - EE


SIGNALS & SYSTEMS

YEAR 2013
Q. 1

Q. 2

ONE MARK

A band-limited signal with a maximum frequency of 5 kHz is to be sampled.


According to the sampling theorem, the sampling frequency which is not valid is
(A) 5 kHz
(B) 12 kHz
(C) 15 kHz
(D) 20 kHz
For a periodic signal v ^ t h = 30 sin 100t + 10 cos 300t + 6 sin ^500t + p/4h, the
fundamental frequency in rad/s
(A) 100
(B) 300
(C) 500

Q. 3

n
i
.
o
c

(D) 1500

Two systems with impulse responses h1 ^ t h and h2 ^ t h are connected in cascade.


Then the overall impulse response of the cascaded system is given by
(A) product of h1 ^ t h and h2 ^ t h
(B) sum of h1 ^ t h and h2 ^ t h
(C) convolution of h1 ^ t h and h2 ^ t h
(D) subtraction of h2 ^ t h from h1 ^ t h

.
a

Q. 4

i
d
o

.n
w

Which one of the following statements is NOT TRUE for a continuous time
causal and stable LTI system?
(A) All the poles of the system must lie on the left side of the jw axis

(B) Zeros of the system can lie anywhere in the s-plane

(C) All the poles must lie within s = 1


(D) All the roots of the characteristic equation must be located on the left side
of the jw axis.
Q. 5

The impulse response of a system is h ^ t h = tu ^ t h. For an input u ^t - 1h, the


output is
2
t ^t - 1h
(A) t u ^ t h
(B)
u ^t - 1h
2
2
(C)

^t - 1h2
u ^t - 1h
2

YEAR 2013
Q. 6

2
(D) t - 1 u ^t - 1h
2

TWO MARKS

The impulse response of a continuous time system is given by h ^ t h = d ^t - 1h + d ^t - 3h


. The value of the step response at t = 2 is
(A) 0
(B) 1
(C) 2
(D) 3

GATE SOLVED PAPER - EE

SIGNALS & SYSTEMS

YEAR 2012
Q. 7

Q. 8

ONE MARK

If x [n] = (1/3) n - (1/2) n u [n], then the region of convergence (ROC) of its z
-transform in the z -plane will be
(A) 1 < z < 3
(B) 1 < z < 1
3
3
2
(C) 1 < z < 3
(D) 1 < z
2
3
The unilateral Laplace transform of f (t) is 2 1
. The unilateral Laplace
s +s+1
transform of tf (t) is
(A) - 2 s
(B) - 2 2s + 1 2
(s + s + 1) 2
(s + s + 1)
(C)

s
(s2 + s + 1) 2

(D)

2s + 1
(s2 + s + 1) 2

n
i
.
o
c

YEAR 2012

TWO MARKS

Q. 9

Let y [n] denote the convolution of h [n] and g [n], where h [n] = (1/2) n u [n] and g [n]
is a causal sequence. If y [0] = 1 and y [1] = 1/2, then g [1] equals
(A) 0
(B) 1/2
(C) 1
(D) 3/2

Q. 10

The Fourier transform of a signal h (t) is H (jw) = (2 cos w) (sin 2w) /w . The value
of h (0) is
(B) 1/2
(A) 1/4
(C) 1
(D) 2

Q. 11

The input x (t) and output y (t) of a system are related as y (t) =
. The system is

.
a
i

d
o

.n
w

# x (t) cos (3t) dt


-3

(A) time-invariant and stable


(B) stable and not time-invariant
(C) time-invariant and not stable
(D) not time-invariant and not stable
YEAR 2011
Q. 12

The Fourier series expansion f (t) = a 0 +

ONE MARK
3

/ an cos nwt + bn sin nwt of

n=1

the periodic signal shown below will contain the following nonzero terms

(A) a 0 and bn, n = 1, 3, 5, ...3


(C) a 0 an and bn, n = 1, 2, 3, ...3

(B) a 0 and an, n = 1, 2, 3, ...3


(D) a 0 and an n = 1, 3, 5, ...3

GATE SOLVED PAPER - EE

Q. 13

SIGNALS & SYSTEMS

Given two continuous time signals x (t) = e-t and y (t) = e-2t which exist for t > 0
, the convolution z (t) = x (t) * y (t) is
(A) e-t - e-2t
(B) e-3t
(C) e+t
(D) e-t + e-2t
YEAR 2011

Q. 14

TWO MARKS

Let the Laplace transform of a function f (t) which exists for t > 0 be F1 (s)
and the Laplace transform of its delayed version f (t - t) be F2 (s). Let F1 * (s)
be the complex conjugate of F1 (s) with the Laplace variable set s = s + jw . If
F (s) F1 * (s)
, then the inverse Laplace transform of G (s) is an ideal
G (s) = 2
F1 (s) 2
(A) impulse d (t)
(C) step function u (t)

Q. 15

(B) delayed impulse d (t - t)


(D) delayed step function u (t - t)

The response h (t) of a linear time invariant system to an impulse d (t), under
initially relaxed condition is h (t) = e-t + e-2t . The response of this system for a
unit step input u (t) is
(A) u (t) + e-t + e-2t
(B) (e-t + e-2t) u (t)
(C) (1.5 - e-t - 0.5e-2t) u (t)
(D) e-t d (t) + e-2t u (t)

Q. 16

Q. 17

a.

i
d

YEAR 2010

in
.
o
c

ONE MARK

For the system 2/ (s + 1), the approximate time taken for a step response to reach
98% of the final value is
(A) 1 s
(B) 2 s
(C) 4 s
(D) 8 s

.
w

o
n

The period of the signal x (t) = 8 sin `0.8pt + p j is


4
(B) 0.8p s
(A) 0.4p s
(D) 2.5 s
(C) 1.25 s

5t

Q. 18

The system represented by the input-output relationship y (t) = # x (t) dt, t > 0
-3
is
(A) Linear and causal
(B) Linear but not causal
(C) Causal but not linear
(D) Neither liner nor causal

Q. 19

The second harmonic component of the periodic waveform given in the figure has
an amplitude of

(A) 0

(B) 1

(C) 2/p

(D)

GATE SOLVED PAPER - EE

SIGNALS & SYSTEMS

YEAR 2010
Q. 20

TWO MARKS

x (t) is a positive rectangular pulse from t =- 1 to t =+ 1 with unit height as


shown in the figure. The value of
transform of x (t)} is.

(A) 2
(C) 4
Q. 21

#- 3

X (w) 2 dw " where X (w) is the Fourier

(B) 2p
(D) 4p

Given the finite length input x [n] and the corresponding finite length output y [n]
of an LTI system as shown below, the impulse response h [n] of the system is

(A) h [n] = {1, 0, 0, 1}


(C) h [n] = {1, 1, 1, 1}
-

.
a
i

d
o

.n
w

Common Data Questions Q.22-23.

n
i
.
o
c

(B) h [n] = {1, 0, 1}


(D) h [n] = {1, 1, 1}
-

Given f (t) and g (t) as show below

Q. 22

g (t) can be expressed as


(A) g (t) = f (2t - 3)

Q. 23

(C) g (t) = f`2t - 3 j


2
The Laplace transform of g (t) is
(A) 1 (e3s - e5s)
s
- 3s
(C) e (1 - e - 2s)
s

YEAR 2009
Q. 24

(B) g (t) = f` t - 3j
2
(D) g (t) = f` t - 3 j
2 2
(B) 1 (e - 5s - e - 3s)
s
(D) 1 (e5s - e3s)
s
ONE MARK

A Linear Time Invariant system with an impulse response h (t) produces output
y (t) when input x (t) is applied. When the input x (t - t) is applied to a system
with impulse response h (t - t), the output will be
(A) y (t)
(B) y (2 (t - t))
(C) y (t - t)
(D) y (t - 2t)

GATE SOLVED PAPER - EE

SIGNALS & SYSTEMS

YEAR 2009
Q. 25

TWO MARKS

A cascade of three Linear Time Invariant systems is causal and unstable. From
this, we conclude that
(A) each system in the cascade is individually causal and unstable
(B) at least on system is unstable and at least one system is causal
(C) at least one system is causal and all systems are unstable
(D) the majority are unstable and the majority are causal

Q. 26

The Fourier Series coefficients of a periodic signal x (t) expressed as


x (t) = / k3=- 3 ak e j2pkt/T are given by a- 2 = 2 - j1, a- 1 = 0.5 + j0.2 , a 0 = j2 ,
a 1 = 0.5 - j0.2 , a 2 = 2 + j1 and ak = 0 for k > 2
Which of the following is true ?
(A) x (t) has finite energy because only finitely many coefficients are non-zero
(B) x (t) has zero average value because it is periodic
(C) The imaginary part of x (t) is constant
(D) The real part of x (t) is even

Q. 27

in
.
o
c

The z-transform of a signal x [n] is given by 4z - 3 + 3z - 1 + 2 - 6z2 + 2z3


It is applied to a system, with a transfer function H (z) = 3z - 1 - 2
Let the output be y [n]. Which of the following is true ?
(A) y [n] is non causal with finite support

.
a
i

(B) y [n] is causal with infinite support


(C) y [n] = 0; n > 3
(D) Re [Y (z)] z = e =- Re [Y (z)] z = e
ji

.n
w

od
- ji

Im [Y (z)] z = e = Im [Y (z)] z = e ; - p # q < p


ji

YEAR 2008
Q. 28

ONE MARK

The impulse response of a causal linear time-invariant system is given as h (t).


Now consider the following two statements :
Statement (I): Principle of superposition holds
Statement (II): h (t) = 0 for t < 0
Which one of the following statements is correct ?
(A) Statements (I) is correct and statement (II) is wrong
(B) Statements (II) is correct and statement (I) is wrong
(C) Both Statement (I) and Statement (II) are wrong
(D) Both Statement (I) and Statement (II) are correct

Q. 29

- ji

A signal e - at sin (wt) is the input to a real Linear Time Invariant system. Given K
and f are constants, the output of the system will be of the form Ke - bt sin (vt + f)
where
(A) b need not be equal to a but v equal to w
(B) v need not be equal to w but b equal to a
(C) b equal to a and v equal to w
(D) b need not be equal to a and v need not be equal to w

GATE SOLVED PAPER - EE

SIGNALS & SYSTEMS

YEAR 2008
Q. 30

TWO MARKS

A system with x (t) and output y (t) is defined by the input-output relation :
y (t) =

- 2t

#- 3x (t) dt

The system will be


(A) Casual, time-invariant and unstable
(B) Casual, time-invariant and stable
(C) non-casual, time-invariant and unstable
(D) non-casual, time-variant and unstable
Q. 31

Q. 32

Q. 33

A signal x (t) = sinc (at) where a is a real constant ^sinc (x) = px h is the input
to a Linear Time Invariant system whose impulse response h (t) = sinc (bt), where
b is a real constant. If min (a, b) denotes the minimum of a and b and similarly,
max (a, b) denotes the maximum of a and b, and K is a constant, which one of
the following statements is true about the output of the system ?
(A) It will be of the form Ksinc (gt) where g = min (a, b)
(B) It will be of the form Ksinc (gt) where g = max (a, b)
(C) It will be of the form Ksinc (at)
(D) It can not be a sinc type of signal
sin (px)

d
o

.
a
i

Let x (t) be a periodic signal with time period T , Let y (t) = x (t - t0) + x (t + t0)
for some t0 . The Fourier Series coefficients of y (t) are denoted by bk . If bk = 0 for
all odd k , then t0 can be equal to
(A) T/8
(B) T/4
(C) T/2
(D) 2T

.n
w

H (z) is a transfer function of a real system. When a signal x [n] = (1 + j) n is the


input to such a system, the output is zero. Further, the Region of convergence
(ROC) of ^1 - 12 z - 1h H(z) is the entire Z-plane (except z = 0 ). It can then be
inferred that H (z) can have a minimum of
(A) one pole and one zero
(B) one pole and two zeros
(C) two poles and one zero
D) two poles and two zeros

Q. 34

n
i
.
o
c

z
Given X (z) =
with z > a , the residue of X (z) zn - 1 at z = a for n $ 0
2
(
z
a
)
will be
(A) an - 1
(B) an
(C) nan
(D) nan - 1

Q. 35

Let x (t) = rect^t - 12 h (where rect (x) = 1 for - 12 # x #


sin (px)

1
2

and zero otherwise.

If sinc (x) = px , then the FTof x (t) + x (- t) will be given by


(B) 2 sinc` w j
(A) sinc` w j
2p
2p
w
w
w
(C) 2 sinc` j cos ` j
(D) sinc` j sin ` w j
2p
2
2p
2

GATE SOLVED PAPER - EE

Q. 36

SIGNALS & SYSTEMS

Given a sequence x [n], to generate the sequence y [n] = x [3 - 4n], which one of the
following procedures would be correct ?
(A) First delay x (n) by 3 samples to generate z1 [n], then pick every 4th sample
of z1 [n] to generate z2 [n], and than finally time reverse z2 [n] to obtain y [n].
(B) First advance x [n] by 3 samples to generate z1 [n], then pick every 4th sample
of z1 [n] to generate z2 [n], and then finally time reverse z2 [n] to obtain y [n]
(C) First pick every fourth sample of x [n] to generate v1 [n], time-reverse v1 [n] to
obtain v2 [n], and finally advance v2 [n] by 3 samples to obtain y [n]
(D) First pick every fourth sample of x [n] to generate v1 [n], time-reverse v1 [n] to
obtain v2 [n], and finally delay v2 [n] by 3 samples to obtain y [n]
YEAR 2007

Q. 37

ONE MARK

Let a signal a1 sin (w1 t + f) be applied to a stable linear time variant system. Let
the corresponding steady state output be represented as a2 F (w2 t + f2). Then
which of the following statement is true?

in
.
o
c

(A) F is not necessarily a Sine or Cosine function but must be periodic


with w1 = w2 .
(B) F must be a Sine or Cosine function with a1 = a2
(C) F must be a Sine function with w1 = w2 and f1 = f2
(D) F must be a Sine or Cosine function with w1 = w2
Q. 38

.
a
i

d
o
n

The frequency spectrum of a signal is shown in the figure. If this is ideally sampled
at intervals of 1 ms, then the frequency spectrum of the sampled signal will be

.
w

GATE SOLVED PAPER - EE

SIGNALS & SYSTEMS

.
a
i

d
o

YEAR 2007
Q. 39

.n
w

n
i
.
o
c

TWO MARKS

A signal x (t) is given by


1, - T/4 < t # 3T/4
x (t) = *- 1, 3T/4 < t # 7T/4
- x (t + T)

Which among the following gives the fundamental fourier term of x (t) ?
(A) 4 cos ` pt - p j
p
T
4
p
p
t
(B) cos `
+ pj
4
2T
4
(C) 4 sin ` pt - p j
p
T
4
p
p
t
(D) sin `
+ pj
4
2T
4

Common Data Questions Q. 40 - 41


A signal is processed by a causal filter with transfer function G (s)
Q. 40

For a distortion free output signal wave form, G (s) must


(A) provides zero phase shift for all frequency
(B) provides constant phase shift for all frequency
(C) provides linear phase shift that is proportional to frequency
(D) provides a phase shift that is inversely proportional to frequency

GATE SOLVED PAPER - EE

Q. 41

SIGNALS & SYSTEMS

G (z) = az - 1 + bz - 3 is a low pass digital filter with a phase characteristics same as


that of the above question if
(B) a =- b
(A) a = b
(D) a = b(- 1/3)

(C) a = b(1/3)
Q. 42

Consider the discrete-time system shown in the figure where the impulse response
of G (z) is g (0) = 0, g (1) = g (2) = 1, g (3) = g (4) = g = 0

This system is stable for range of values of K


(A) [- 1, 12 ]
(B) [- 1, 1]
1
(C) [- 2 , 1]
(D) [- 12 , 2]
Q. 43

If u (t), r (t) denote the unit step and unit ramp functions respectively and
u (t) * r (t) their convolution, then the function u (t + 1) * r (t - 2) is given by
(A) 12 (t - 1) u (t - 1)
(B) 12 (t - 1) u (t - 2)
(C)

Q. 44

1
2

(t - 1) 2 u (t - 1)
-1

in
.
o
c

(D) None of the above


-2

X (z) = 1 - 3z , Y (z) = 1 + 2z are Z transforms of two signals x [n], y [n]


respectively. A linear time invariant system has the impulse response h [n] defined
by these two signals as h [n] = x [n - 1] * y [n] where * denotes discrete time
convolution. Then the output of the system for the input d [n - 1]
(A) has Z-transform z - 1 X (z) Y (z)
(B) equals d [n - 2] - 3d [n - 3] + 2d [n - 4] - 6d [n - 5]

d
o
n

.
a
i

.
w

(C) has Z-transform 1 - 3z - 1 + 2z - 2 - 6z - 3


(D) does not satisfy any of the above three

YEAR 2006

ONE MARK

Q. 45

The following is true


(A) A finite signal is always bounded
(B) A bounded signal always possesses finite energy
(C) A bounded signal is always zero outside the interval [- t0, t0] for some t0
(D) A bounded signal is always finite

Q. 46

x (t) is a real valued function of a real variable with period T . Its trigonometric
Fourier Series expansion contains no terms of frequency w = 2p (2k) /T; k = 1, 2g
Also, no sine terms are present. Then x (t) satisfies the equation
(A) x (t) =- x (t - T)
(B) x (t) = x (T - t) =- x (- t)
(C) x (t) = x (T - t) =- x (t - T/2)
(D) x (t) = x (t - T) = x (t - T/2)

Q. 47

A discrete real all pass system has a pole at z = 2+30% : it, therefore
(A) also has a pole at 12 +30%
(B) has a constant phase response over the z -plane: arg H (z) = constant

GATE SOLVED PAPER - EE

SIGNALS & SYSTEMS

constant
(C) is stable only if it is anti-causal
(D) has a constant phase response over the unit circle: arg H (eiW) = constant
YEAR 2006
Q. 48

Q. 49

TWO MARKS

x [n] = 0; n < - 1, n > 0, x [- 1] =- 1, x [0] = 2 is the input and


y [n] = 0; n < - 1, n > 2, y [- 1] =- 1 = y [1], y [0] = 3, y [2] =- 2 is the output of a
discrete-time LTI system. The system impulse response h [n] will be
(A) h [n] = 0; n < 0, n > 2, h [0] = 1, h [1] = h [2] =- 1
(B) h [n] = 0; n < - 1, n > 1, h [- 1] = 1, h [0] = h [1] = 2
(C) h [n] = 0; n < 0, n > 3, h [0] =- 1, h [1] = 2, h [2] = 1
(D) h [n] = 0; n < - 2, n > 1, h [- 2] = h [1] = h [- 1] =- h [0] = 3

n
i
.
o
c

n
The discrete-time signal x [n]
X (z) = / n3= 0 3 z2n , where
2+n
transform-pair relationship, is orthogonal to the signal
n
(A) y1 [n] ) Y1 (z) = / n3= 0 ` 2 j z - n
3

.
a
i

(B) y2 [n] ) Y2 (z) = / n3= 0 (5n - n) z - (2n + 1)

d
o

(C) y3 [n] ) Y3 (z) = / n3=- 3 2 - n z - n

.n
w

denotes a

(D) y4 [n] ) Y4 (z) = 2z - 4 + 3z - 2 + 1


Q. 50

A continuous-time system is described by y (t) = e - x (t) , where y (t) is the output


and x (t) is the input. y (t) is bounded
(A) only when x (t) is bounded
(B) only when x (t) is non-negative

(C) only for t # 0 if x (t) is bounded for t $ 0


(D) even when x (t) is not bounded
Q. 51

The running integration, given by y (t) =

#- 3 x (t') dt'

(A) has no finite singularities in its double sided Laplace Transform Y (s)
(B) produces a bounded output for every causal bounded input
(C) produces a bounded output for every anticausal bounded input
(D) has no finite zeroes in its double sided Laplace Transform Y (s)
YEAR 2005
Q. 52

TWO MARKS

For the triangular wave from shown in the figure, the RMS value of the voltage
is equal to

GATE SOLVED PAPER - EE

SIGNALS & SYSTEMS

1
6

(A)

(B)

(C) 1
3
Q. 53

Q. 54

Q. 55

(D)

2
The Laplace transform of a function f (t) is F (s) = 5s 2+ 23s + 6 as t " 3, f (t)
s (s + 2s + 2)
approaches
(A) 3
(B) 5
(C) 17
(D) 3
2
The Fourier series for the function f (x) = sin2 x is
(A) sin x + sin 2x
(B) 1 - cos 2x
(C) sin 2x + cos 2x
(D) 0.5 - 0.5 cos 2x

If u (t) is the unit step and d (t) is the unit impulse function, the inverse z
-transform of F (z) = z +1 1 for k > 0 is
(A) (- 1) k d (k)

(B) d (k) - (- 1) k

(C) (- 1) k u (k)

(D) u (k) - (- 1) k

Q. 57

in
.
o
c

.
a
i

YEAR 2004
Q. 56

1
3
2
3

TWO MARKS

The rms value of the resultant current in a wire which carries a dc current of 10
A and a sinusoidal alternating current of peak value 20 is
(A) 14.1 A
(B) 17.3 A
(C) 22.4 A
(D) 30.0 A

d
o
n

.
w

The rms value of the periodic waveform given in figure is

(A) 2 6 A

(B) 6 2 A

(C)

(D) 1.5 A

4/3 A

***********

GATE SOLVED PAPER - EE

SIGNALS & SYSTEMS

SOLUTION
Sol. 1

Option (A) is correct.


Given, the maximum frequency of the band-limited signal
fm = 5 kHz
According to the Nyquist sampling theorem, the sampling frequency must be
greater than the Nyquist frequency which is given as
fN = 2fm = 2 # 5 = 10 kHz
So, the sampling frequency fs must satisfy

n
i
.
o
c

fs $ fN
fs $ 10 kHz
only the option (A) does not satisfy the condition therefore, 5 kHz is not a
valid sampling frequency.
Sol. 2

Option (A) is correct.


Given, the signal

d
o

.
a
i

v ^ t h = 30 sin 100t + 10 cos 300t + 6 sin ^500t + p4 h


So we have
w1 = 100 rad/s ; w2 = 300 rad/s and w3 = 500 rad/s
Therefore, the respective time periods are
T1 = 2p = 2p sec
w1 100
T2 = 2p = 2p sec
300
w2
T3 = 2p sec
500

Sol. 3

.n
w

So, the fundamental time period of the signal is


LCM ^2p, 2p, 2ph
L.C.M. ^T1, T2 T3h =
HCF ^100, 300, 500h
or,
T0 = 2p
100
Thus, the fundamental frequency in rad/sec is
w0 = 2p = 100 rad/s
10
Option (C) is correct.
If the two systems with impulse response h1 ^ t h and h2 ^ t h are connected in
cascaded configuration as shown in figure, then the overall response of the system
is the convolution of the individual impulse responses.

GATE SOLVED PAPER - EE

SIGNALS & SYSTEMS

Sol. 4

Option (C) is correct.


For a system to be casual, the R.O.C of system transfer function H ^s h which is
rational should be in the right half plane and to the right of the right most pole.
For the stability of LTI system. All poles of the system should lie in the
left half of S -plane and no repeated pole should be on imaginary axis. Hence,
options (A), (B), (D) satisfies both stability and causality an LTI system.
But, Option (C) is not true for the stable system as, S = 1 have one pole
in right hand plane also.

Sol. 5

Option (C) is correct.


Given, the input
x ^ t h = u ^t - 1h
Its Laplace transform is
-s
X ^s h = e
s
The impulse response of system is given

h^t h = t u^t h
Its Laplace transform is
H ^s h = 12
s
Hence, the overall response at the output is
-s
Y ^s h = X ^s h H ^s h = e 3
s
Its inverse Laplace transform is
^t - 1h2
y^t h =
u ^t - 1h
2
Option (B) is correct.
Given, the impulse response of continuous time system

in
.
o
c

.
a
i

Sol. 6

d
o
n

.
w

h ^ t h = d ^t - 1h + d ^t - 3h
From the convolution property, we know
x ^ t h * d ^t - t 0h = x ^t - t 0h
So, for the input
x ^ t h = u ^ t h (Unit step fun n )
The output of the system is obtained as

y^t h = u^t h * h^t h

= u ^ t h * 6d ^t - 1h + d ^t - 3h@
= u ^t - 1h + u ^t - 3h

At t = 2
Sol. 7

y ^2 h = u ^2 - 1h + u ^2 - 3h = 1

Option (C) is correct.


n
n
x [ n] = b 1 l - b 1 l u [ n]
3
2
n
-n
n
= b 1 l u [n] + b 1 l u [- n - 1] - b 1 l u (n)
3
3
2
Taking z -transform

X 6z @ =

/
n =- 3

1 n -n
b 3 l z u [ n] +

/
n =- 3

1 -n -n
b 3 l z u [ - n - 1]

GATE SOLVED PAPER - EE

SIGNALS & SYSTEMS

1 n -n
b 2 l z u [ n]
n =- 3
-1
3
3
1 n z-n +
1 -n z-n 1 n -n
=
b3l
b3l
b2l z
n=0
n =- 3
n=0
-

/ b 31z l + / b 13 z l

- /b 1 l
2z
m=1
n=0
1 44 2
4
4
3
1
4
4
2
4
43
II
III

n=0

14
42
4
43
I

Taking m =- n

1 < 1 or z > 1
3
3z
1
Series II converges if z < 1 or z < 3
3
Series III converges if 1 < 1 or z > 1
2
2z
Region of convergence of X (z) will be intersection of above three
So,
ROC : 1 < z < 3
2
Option (D) is correct.
Using s -domain differentiation property of Laplace transform.
Series I converges if

Sol. 8

If

f (t)

a.

i
d
o

F (s)

dF (s)
ds
d
1
2s + 1
L [tf (t)] =
=
ds ;s2 + s + 1E (s2 + s + 1) 2
tf (t)

So,
Sol. 9

n
i
.
o
c

.n
w

Option (A) is correct.


Convolution sum is defined as

For causal sequence,

y [n] = h [n] * g [n] =


y [ n] =

/ h [n] g [n - k]
k =- 3

/ h [n] g [n - k]
k=0

y [n] = h [n] g [n] + h [n] g [n - 1] + h [n] g [n - 2] + .....

For n = 0 ,

y [0] = h [0] g [0] + h [1] g [- 1] + ...........


= h [ 0] g [ 0 ]
g [- 1] = g [- 2] = ....0
...(i)
= h [ 0] g [ 0 ]

For n = 1,

y [1] = h [1] g [1] + h [1] g [0] + h [1] g [- 1] + ....


= h [ 1] g [ 1 ] + h [ 1 ] g [ 0 ]
1 = 1 g [1] + 1 g [0]
2
2
2

1
h [1] = b 1 l = 1
2
2

1 = g [1] + g [0]
From equation (i),

g [1] = 1 - 1 = 0

So,
Sol. 10

g [1] = 1 - g [0]
y [ 0] 1
g [0] =
= =1
h [ 0] 1

Option (C) is correct.


(2 cos w) (sin 2w)
= sin 3w + sin w
w
w
w
We know that inverse Fourier transform of sin c function is a rectangular function.
H (jw) =

GATE SOLVED PAPER - EE

SIGNALS & SYSTEMS

So, inverse Fourier transform of H (jw)


h (t) = h1 (t) + h2 (t)
h (0) = h1 (0) + h2 (0) = 1 + 1 = 1
2 2
Sol. 11

in
.
o
c

Option (D) is correct.


y (t) =
Time invariance :
Let,

# x (t) cos (3t) dt


-3

.
a
i

x (t) = d (t)
t

# d (t) cos (3t) dt

d
o
n#

y (t) =

-3

= u (t) cos (0) = u (t)

For a delayed input (t - t 0) output is

.
w

y (t, t 0) =

Delayed output

d (t - t 0) cos (3t) dt = u (t) cos (3t 0)

-3

y (t - t 0) = u (t - t 0)
y (t, t 0) ! y (t - t 0)
System is not time invariant.
Stability :
Consider a bounded input x (t) = cos 3t

y (t) =

-3

cos2 3t =

1 - cos 6t = 1
2
2
-3

# 1dt - 12 # cos 6t dt
t

-3

-3

As t " 3, y (t) " 3 (unbounded)


System is not stable.
Sol. 12

Option (D) is correct.

f (t) = a 0 +

/ (an cos wt + bn sin nwt)

n=1

The given function f (t) is an even function, therefore bn = 0

GATE SOLVED PAPER - EE

SIGNALS & SYSTEMS

f (t) is a non zero average value function, so it will have a non-zero value of a 0
T/2
a 0 = 1 # f (t) dt (average value of f (t))
^T/2h 0
an is zero for all even values of n and non zero for odd n
T
an = 2 # f (t) cos (nwt) d (wt)
T 0
So, Fourier expansion of f (t) will have a 0 and an , n = 1, 3, 5f3
Sol. 13

Option (A) is correct.


x (t) = e-t
Laplace transformation
X (s) =

1
s+1

y (t) = e-2t
Y (s) = 1
s+2
Convolution in time domain is equivalent to multiplication in frequency domain.

n
i
.
o
c

.
a
i

z (t) = x (t) ) y (t)

Z (s) = X (s) Y (s) = b 1 lb 1 l


s+1 s+2
By partial fraction and taking inverse Laplace transformation, we get
Z (s) = 1 - 1
s+1 s+2

d
o

.n
w

z (t) = e-t - e-2t

Sol. 14

Option (D) is correct.

f (t)

F1 (s)

e-st F1 (s) = F2 (s)


F (s) F 1)(s) e-st F1 (s) F 1)(s)
G (s) = 2
=
F1 (s) 2
F1 (s) 2
e-sE F1 (s) 2
)
2
=
"a F1 (s) F 1 (s) = F1 (s)
F1 (s) 2
= e-st
Taking inverse Laplace transform

f (t - t)

g (t) = L - 1 [e-st] = d (t - t)
Sol. 15

Option (C) is correct.


h (t) = e-t + e-2t
Laplace transform of h (t) i.e. the transfer function
H (s) = 1 + 1
s+1 s+2
For unit step input
or
Output,
By partial fraction

r (t) = m (t)
R (s) = 1
s
Y (s) = R (s) H (s) = 1 : 1 + 1 D
s s+1 s+2

GATE SOLVED PAPER - EE

SIGNALS & SYSTEMS

Y (s) = 3 - 1 - b 1 l 1
2s s + 1
s+2 2
Taking inverse Laplace
e-2t u (t)
y (t) = 3 u (t) - e-t u (t) 2
2
= u (t) 61.5 - e-t - 0.5e-2t@
Sol. 16

Option (C) is correct.


System is given as
2
(s + 1)
R (s) = 1
s

H (s) =
Step input

2
1 =2- 2
(s + 1) b s l s (s + 1)

Y (s) = H (s) R (s) =

Output

Taking inverse Laplace transform


y (t) = (2 - 2e- t) u (t)
Final value of y (t),

in
.
o
c

yss (t) = lim y (t) = 2


t"3

Let time taken for step response to reach 98% of its final value is ts .
So,

.
a
i

2 - 2e- ts = 2 # 0.98
0.02 = e- ts
ts = ln 50 = 3.91 sec.
Sol. 17

d
o
n

Option (D) is correct.


Period of x (t),

.
w

T = 2p = 2 p = 2.5 sec
w
0.8 p

Sol. 18

Option (B) is correct.


Input output relationship

y (t) =

5t

#- 3x (t) dt,

t>0

Causality :
y (t) depends on x (5t), t > 0 system is non-causal.
For example t = 2
y (2) depends on x (10) (future value of input)
Linearity :
Output is integration of input which is a linear function, so system is linear.
Sol. 19

Option (A) is correct.


Fourier series of given function
3

x (t) = A0 + / an cos nw0 t + bn sin nw0 t


n=1

So,

a x (t) =- x (t) odd function


A0 = 0
an = 0
bn = 2
T

#0 x (t) sin nw0 t dt

GATE SOLVED PAPER - EE

SIGNALS & SYSTEMS

T /2
T
= 2 = # (1) sin nw0 t dt + # (- 1) sin nw0 t dt G
T 0
T /2
T
/
T
2
= 2 =c cos nw0 t m - c cos nw0 t m G
T
- nw0 0
- nw0 T/2
2
=
(1 - cos np) + (cos 2np - cos np)@
nw0 T 6
= 2 61 - (- 1) n @
np
4 , n odd
bn = * np
0 , n even
So only odd harmonic will be present in x (t)
For second harmonic component (n = 2) amplitude is zero.

Sol. 20

Option (D) is correct.


By parsvals theorem
1 3 X (w) 2 dw =
#
2p - 3

#- 3

Sol. 21

#- 3

x (t) dt

.
a
i

X (w) 2 dw = 2p # 2 = 4p

Option (C) is correct.


Given sequences

d
o

.n
w

n
i
.
o
c

3 2

x [n] = {1, - 1}, 0 # n # 1


-

y [n] = {1, 0, 0, 0, - 1}, 0 # n # 4


If impulse response is h [n] then

y [ n ] = h [ n] * x [ n]
Length of convolution (y [n]) is 0 to 4, x [n] is of length 0 to 1 so length of h [n]
will be 0 to 3.
Let

h [n] = {a, b, c, d}
-

Convolution

y [n] = {a, - a + b, - b + c, - c + d, - d}
By comparing
a =1
-a + b = 0 & b = a = 1

So,

-b + c = 0 & c = b = 1
-c + d = 0 & d = c = 1
h [n] = {1, 1, 1, 1}
-

GATE SOLVED PAPER - EE

Sol. 22

SIGNALS & SYSTEMS

Option (D) is correct.


We can observe that if we scale f (t) by a factor of 1 and then shift, we will get
2
g (t).
First scale f (t) by a factor of 1
2
g1 (t) = f (t/2)

g (t) = g1 (t - 3) = f` t - 3 j
2

Shift g1 (t) by 3,

g (t) = f` t - 3 j
2 2
Sol. 23

in
.
o
c

.
a
i

Option (C) is correct.


g (t) can be expressed as

d
o
n

g (t) = u (t - 3) - u (t - 5)
By shifting property we can write Laplace transform of g (t)
- 3s
G (s) = 1 e - 3s - 1 e - 5s = e (1 - e - 2s)
s
s
s

Sol. 24

.
w

Option (D) is correct.


L
Let
x (t)
X (s)
L
y (t)
Y (s)
L
h (t)
H (s)
So output of the system is given as

Y (s) = X (s) H (s)

Now for input


So now output is

x (t - t)

h (t - t)

Y' (s) = e

e - st X (s)
e
- st

- st

(shifting property)

H (s)

X (s) $ e - ts H (s)

= e - 2st X (s) H (s) = e - 2st Y (s)


y' (t) = y (t - 2t)
Sol. 25

Option (B) is correct.


Let three LTI systems having response H1 (z), H2 (z) and H 3 (z) are
Cascaded as showing below

Assume

H1 (z) = z2 + z1 + 1 (non-causal)
H2 (z) = z3 + z2 + 1 (non-causal)

GATE SOLVED PAPER - EE

SIGNALS & SYSTEMS

Overall response of the system


H (z) = H1 (z) H2 (z) H3 (z)
H (z) = (z2 + z1 + 1) (z3 + z2 + 1) H3 (z)
To make H (z) causal we have to take H3 (z) also causal.
H3 (z) = z - 6 + z - 4 + 1

Let

= (z2 + z1 + 1) (z3 + z2 + 1) (z - 6 + z - 4 + 1)
H (z) " causal
Similarly to make H (z) unstable atleast one of the system should be unstable.
Sol. 26

Option (C) is correct.


Given signal
x (t) =

/ ak e j2pkt/T

n
i
.
o
c

k =- 3

Let w0 is the fundamental frequency of signal x (t)


x (t) =

/ ak e jkw t
0

k =- 3

a.

a 2p = w0
T

x (t) = a - 2 e - j2w t + a - 1 e - jw t + a0 + a1 e jw t + a2 e j2w t


0

= (2 - j) e

- 2jw0 t

di

no

+ (0.5 + 0.2j) e

- jw0 t

+ 2j +

+ (0.5 - 0.2) e jw t + (2 + j) e j2w t


0

= 2 6e - j2w t + e j2w t @ + j 6e j2w t - e - j2w t @ +


0

0.5 6e jw t + e - jw t @ - 0.2j 6e+ jw t - e - jw t @ + 2j


= 2 (2 cos 2w0 t) + j (2j sin 2w0 t) + 0.5 (2 cos w0 t) -

.
w

0.2j (2j sin w0 t) + 2j


= 64 cos 2w0 t - 2 sin 2w0 t + cos w0 t + 0.4 sin w0 t @ + 2j
Im [x (t)] = 2 (constant)

Sol. 27

Option (A) is correct.


Z-transform of x [n] is
X (z) = 4z - 3 + 3z - 1 + 2 - 6z2 + 2z3
Transfer function of the system
H (z) = 3z - 1 - 2
Output
Y (z) = H (z) X (z)
Y (z) = (3z - 1 - 2) (4z - 3 + 3z - 1 + 2 - 6z2 + 2z3)
= 12z -4 + 9z -2 + 6z -1 - 18z + 6z2 - 8z -3 - 6z -1 - 4 + 12z2 - 4z3
= 12z - 4 - 8z - 3 + 9z - 2 - 4 - 18z + 18z2 - 4z3
Or sequence y [n] is
y [n] = 12d [n - 4] - 8d [n - 3] + 9d [n - 2] - 4d [n] 18d [n + 1] + 18d [n + 2] - 4d [n + 3]
y [n] =
Y 0, n < 0
So y [n] is non-causal with finite support.

Sol. 28

Option (D) is correct.


Since the given system is LTI, So principal of Superposition holds due to linearity.
For causal system h (t) = 0 , t < 0
Both statement are correct.

GATE SOLVED PAPER - EE

Sol. 29

SIGNALS & SYSTEMS

Option (C) is correct.


For an LTI system output is a constant multiplicative of input with same frequency.
input g (t) = e - at sin (wt)

Here

output y (t) = Ke - bt sin (vt + f)


Output will be in form of Ke - at sin (wt + f)
So
\= b, v = w
Sol. 30

Option (D) is correct.


Input-output relation
y (t) =

- 2t

#- 3x (t) dt

Causality :
Since y (t) depends on x (- 2t), So it is non-causal.
Time-variance :
y (t) =

- 2t

#- 3x (t - t0) dt =Y y (t - t0)

So this is time-variant.
Stability :
Output y (t) is unbounded for an bounded input.
For example
x (t) = e - t (bounded)

Let

y (t) =
Sol. 31

in
.
o
c

.
a
i

- 2t

- t - 2t

#- 3e- t dt = 8-e 1 B- 3 $ Unbounded

d
o
n

Option (A) is correct.


Output y (t) of the given system is

.
w

y (t) = x (t) ) h (t)


Or
Y (jw) = X (jw) H (jw)
Given that,
x (t) = sinc (at) and h (t) = sinc (bt)
Fourier transform of x (t) and h (t) are
X (jw) = F [x (t)] = p rect` w j, - a < w < a
a
2a
p
H (jw) = F [h (t)] = rect` w j, - b < w < b
b
2b

So,

2
Y (jw) = p rect` w j rect` w j
ab
2a
2b

Y (jw) = K rect ` w j
2g

Where
And
Sol. 32

Option (B) is correct.

g = min (a, b)
y (t) = K sinc (gt)

GATE SOLVED PAPER - EE

SIGNALS & SYSTEMS

Let ak is the Fourier series coefficient of signal x (t)


Given
y (t) = x (t - t0) + x (t + t0)
Fourier series coefficient of y (t)
bk = e - jkwt ak + e jkwt ak
bk = 2ak cos kwt0
bk = 0 (for all odd k )
kwt0 = p , k " odd
2
k 2p t0 = p
2
T
T
t0 =
4
0

For k = 1,
Sol. 33

Option ( ) is correct.

Sol. 34

Option (D) is correct.

n
i
.
o
c

z
, z >a
(z - a) 2
Residue of X (z) zn - 1 at z = a is
= d (z - a) 2 X (z) zn - 1 z = a
dz
z
= d (z - a) 2
zn - 1
2
dz
(z - a)
z=a
n-1
n
d
z
=
= nz z = a = nan - 1
dz z = a
Option (C) is correct.
Given signal
x (t) = rect `t - 1 j
2
1, - 1 # t - 1 # 1 or 0 # t # 1
2
2 2
So,
x (t) = *
0, elsewhere
Similarly
x (- t) = rect`- t - 1 j
2
1, - 1 # - t - 1 # 1 or - 1 # t # 0
2
2 2
x (- t) = *
0, elsewhere
X (z) =

Given that

Sol. 35

.
a
i

d
o

.n
w

F [x (t) + x (- t)] =
=

#- 3 x (t) e- jwt dt + #- 3 x (- t) e- jwt dt


3

#0 (1) e- jwt dt + #- 1 (1) e- jwt dt


1

- jw t
- jw t
= ; e E + ; e E = 1 (1 - e - jw) + 1 (e jw - 1)
jw
jw
- jw 0
- jw - 1
- j w /2
j w /2
=e
(e jw/2 - e - jw/2) + e
(e jw/2 - e - jw/2)
jw
jw

(e jw/2 - e - jw/2) (e - jw/2 + e jw/2)


jw
2
w
= sin ` j $ 2 cos ` w j = 2 cos w sinc` w j
w
2
2
2
2p
=

GATE SOLVED PAPER - EE

Sol. 36

SIGNALS & SYSTEMS

Option (B) is correct.


In option (A)
z1 [n] = x [n - 3]
z2 [n] = z1 [4n] = x [4n - 3]
y [n] = z2 [- n] = x [- 4n - 3] =
Y x [3 - 4n]
In option (B)
z1 [n] = x [n + 3]
z2 [n] = z1 [4n] = x [4n + 3]
y [n] = z2 [- n] = x [- 4n + 3]
In option (C)
v1 [n] = x [4n]
v2 [n] = v1 [- n] = x [- 4n]
y [n] = v2 [n + 3] = x [- 4 (n + 3)] =
Y x [3 - 4n]
In option (D)
v1 [n] = x [4n]

in
.
o
c

v2 [n] = v1 [- n] = x [- 4n]
y [n] = v2 [n - 3] = x [- 4 (n - 3)] =
Y x [3 - 4n]
Sol. 37

Option ( ) is correct.
The spectrum of sampled signal s (jw) contains replicas of U (jw) at frequencies
!nfs .
Where
n = 0, 1, 2.......
1
fs = 1 =
= 1 kHz
Ts
1 m sec

.
a
i

Sol. 38

.
w

d
o
n

Option (D) is correct.


For an LTI system input and output have identical wave shape (i.e. frequency of
input-output is same) within a multiplicative constant (i.e. Amplitude response

GATE SOLVED PAPER - EE

SIGNALS & SYSTEMS

is constant)
So F must be a sine or cosine wave with w1 = w2
Sol. 39

Option (C) is correct.


Given signal has the following wave-form

n
i
.
o
c

.
a
i

Function x(t) is periodic with period 2T and given that

x (t) =- x (t + T) (Half-wave symmetric)


So we can obtain the fourier series representation of given function.
Sol. 40

d
o

.n
w

Option (C) is correct.


Output is said to be distortion less if the input and output have identical wave
shapes within a multiplicative constant. A delayed output that retains input
waveform is also considered distortion less.
Thus for distortion less output, input-output relationship is given as

y (t) = Kg (t - td )
Taking Fourier transform.

Y (w) = KG (w) e - jwt = G (w) H (w)


H (w) & transfer function of the system
d

So,
H (w) = Ke - jwt
Amplitude response H (w) = K
Phase response,
qn (w) =- wtd
For distortion less output, phase response should be proportional to frequency.
d

Sol. 41

Option (A) is correct.


G (z) z = e = ae- jw + be- 3jw
for linear phase characteristic a = b .
jw

Sol. 42

Option (A) is correct.


System response is given as
G (z)
H (z) =
1 - KG (z)
g [n] = d [n - 1] + d [n - 2]
G (z) = z - 1 + z - 2
So

H (z) =

(z - 1 + z - 2)
= 2 z+1
-1
-2
z - Kz - K
1 - K (z + z )

GATE SOLVED PAPER - EE

SIGNALS & SYSTEMS

For system to be stable poles should lie inside unit circle.


z #1
2
z = K ! K + 4K # 1 K ! K2 + 4K # 2
2

K 2 + 4K # 2 - K
K2 + 4K # 4 - 4K + K2
8K # 4
K # 1/2
Sol. 43

Option (C) is correct.


Given Convolution is,
h (t) = u (t + 1) ) r (t - 2)
Taking Laplace transform on both sides,
H (s) = L [h (t)] = L [u (t + 1)] ) L [r (t - 2)]
We know that, L [u (t)] = 1/s
(Time-shifting property)
L [u (t + 1)] = es c 12 m
s
and
L [r (t)] = 1/s2
(Time-shifting property)
L [r (t - 2) = e - 2s c 12 m
s
So
H (s) = ;es ` 1 jE;e - 2s c 12 mE
s
s
H (s) = e - s c 13 m
s
Taking inverse Laplace transform
h (t) = 1 (t - 1) 2 u (t - 1)
2

in
.
o
c

.
a
i

Sol. 44

d
o
n

.
w

Option (C) is correct.


Impulse response of given LTI system.

h [n] = x [n - 1] ) y [n]
Taking z -transform on both sides.

H (z) = z - 1 X (z) Y (z)


We have X (z) = 1 - 3z - 1 and Y (z) = 1 + 2z - 2
So

a x [n - 1]

z - 1 x (z)

H (z) = z - 1 (1 - 3z - 1) (1 + 2z - 2)
Output of the system for input u [n] = d [n - 1] is ,
y (z) = H (z) U (z)

U [n]

U (z) = z - 1

So
Y (z) = z - 1 (1 - 3z - 1) (1 + 2z - 2) z - 1
= z - 2 (1 - 3z - 1 + 2z - 2 - 6z - 3) = z - 2 - 3z - 3 + 2z - 4 - 6z - 5
Taking inverse z-transform on both sides we have output.
y [n] = d [n - 2] - 3d [n - 3] + 2d [n - 4] - 6d [n - 5]
Sol. 45

Option (B) is correct.


A bounded signal always possesses some finite energy.
E =

t0

#- t

g (t) 2 dt < 3

GATE SOLVED PAPER - EE

Sol. 46

SIGNALS & SYSTEMS

Option (C) is correct.


Trigonometric Fourier series is given as
3

x (t) = A0 + / an cos nw0 t + bn sin nw0 t


n=1

Since there are no sine terms, so bn = 0


T
bn = 2 # x (t) sin nw0 t dt
T0 0
0

T /2
x (t) sin nw0 t dt +
= 2 =#
T0 0
0

#T /2 x (t) sin nw0 t dt G


0

Where t = T - t & dt =- dt
T /2
T
x (T - t) sin nw0 (T - t) (- dt)+ # x (t) sin nw0 t dt E
= 2 ;#
T0 T
T /2
0

TO

= 2 ; # x (T - t) sin n` 2p T - t j dt ++ # x (t) sin nw0 t dt E


T0 T /2
T
T /2

n
i
. #
o
c

T
T
= 2 ; # x (T - t) sin (2np - nw0) dt+ # x (t) sin nw0 t dt E
T0 T /2
T /2
0

T
= 2 ;- # x (T - t) sin (nw0 t) dt + +
T0
T /2

.
a
i

T0

T0 /2

x (t) sin nw0 t dt E

bn = 0 if
x (t) = x (T - t)
From half wave symmetry we know that if
x (t) =- x`t ! T j
2
Then Fourier series of x (t) contains only odd harmonics.

d
o

Sol. 47

.n
w

Option (C) is correct.


Z -transform of a discrete all pass system is given as
-1
)
H (z) = z - z-0 1
1 - z0 z
It has a pole at z 0 and a zero at 1/z)
0.
Given system has a pole at

z = 2+30% = 2

( 3 + j)
= ( 3 + j)
2

system is stable if z < 1 and for this it is anti-causal.


Sol. 48

Option (A) is correct.


According to given data input and output Sequences are
x [n] = {- 1, 2}, - 1 # n # 0
y [n] = {- 1, 3, - 1, - 2}, - 1 # n # 2
-

GATE SOLVED PAPER - EE

SIGNALS & SYSTEMS

If impulse response of system is h [n] then output


y [ n] = h [ n] ) x [ n]
Since length of convolution (y [n]) is - 1 to 2, x [n] is of length - 1 to 0 so length
of h [n] is 0 to 2.
h [n] = {a, b, c}
-

Let
Convolution

y [n] = {- a, 2a - b, 2b - c, 2c}
y [n] = {- 1, 3, - 1, - 2}
-

Sol. 49

So, a = 1
2a - b = 3 & b =- 1
2a - c =- 1 & c =- 1
Impulse response h [n] = "1, - 1, - 1,
Option ( ) is correct.

Sol. 50

Option (D) is correct.

in
.
o
c

.
a
i

Output
y (t) = e - x (t)
If x (t) is unbounded, x (t) " 3

d
o
n

y (t) = e - x (t) " 0 (bounded)


So y (t) is bounded even when x (t) is not bounded.
Sol. 51

.
w#

Option (B) is correct.


Given

y (t) =

x (t') dt'

-3

Laplace transform of y (t)


X (s)
, has a singularity at s = 0
Y (s) =
s

# x (t') dt' is always bounded.

For a causal bounded input, y (t)=


Sol. 52

-3

Option (A) is correct.


RMS value is given by
1
T

Vrms =
Where

#0

V2 (t) dt

2
T
` T j t, 0 # t # 2

V (t) = *
So

1
T

#0

0,

T <t#T
2

T /2
2
V 2 (t) dt = 1 = # ` 2t j dt +
T 0
T

= 1 $ 42
T T

#0

T /2

#T/2 (0) dt G
T /2

3
t2 dt = 43 ; t E
T 3 0

3
= 43 # T = 1
6
24
T

GATE SOLVED PAPER - EE

SIGNALS & SYSTEMS

Vrms =
Sol. 53

1 V
6

Option (A) is correct.


By final value theorem
lim f (t) = lim s F (s) = lim s

t"3

Sol. 54

s"0

s"0

Option (D) is correct.

(5s2 + 23s + 6)
= 6 =3
2
s (s2 + 2s + 2)

f (x) = sin2 x = 1 - cos 2x


2
= 0.5 - 0.5 cos 2x
3

f (x) = A0 + / an cos nw0 x + bn sin nw0 x


n=1

f (x) = sin2 x is an even function so bn = 0


A0 = 0.5
- 0.5, n = 1
an = )
0
, otherwise
2
p
w0 =
= 2p = 2
T0
T
Sol. 55

Option (B) is correct.

d
o

.n
w

f (k) = d (k) - (- 1) k
Z
1
(- 1) k
1 + z- 1

so,
Thus

Sol. 57

.
a
i

F (z) = 1 = 1 - z = 1 - 1 - 1
z+1
z+1
1+z

Z-transform

Sol. 56

n
i
.
o
c

Option (B) is correct.


Total current in wire

I = 10 + 20 sin wt
Irms =

(10) 2 +

Option (A) is correct.


Root mean square value is given as

(20) 2
= 17.32 A
2

1 T I2 (t) dt
#
T 0
- 12 t, 0 # t < T
2
I (t) = * ` T j
6,
T/2 < t # T
Irms =

From the graph,

So

1
T

#0

T /2
2
I2 dt = 1 = # ` - 12t j dt +
T 0
T

#T/2 (6) 2 dt G

T /2

t3
= 1 e 144
+ 36 6t @TT/2 o
;
T T2 3 E0
T3 + 36 T
= 1 ; 144
c
` 2 jE
2
T T 24 m
= 1 [6T + 18T] = 24
T
Irms =

24 = 2 6 A

***********

Vous aimerez peut-être aussi